Bài toán (IMO Shortlisted 2002): Cho trước đường tròn (O) và hai điểm A, B sao cho AB tiếp xúc với đường tròn (O) tại B. Lấy điểm C không nằm trên (O) sao cho AC cắt (O) tại hai điểm phân biệt, dựng đường tròn (w) tiếp xúc AC tại C, tiếp xúc với (O) tại D sao cho B, D nằm về hai phía của đường thẳng AC Chứng minh tâm đường tròn ngoại tiếp tam giác BCD nằm trên (ABC).
Lời giải:
Gọi I J thứ tự là tâm của (w) và (BCD), t là tiếp tuyến chung tại D của (O) và (w).
Từ giả thiết suy ra O, J nằm trên trung trực BD và I, J nằm trên trung trực CD suy ra
- Phép đối xứng trục OJ biến BA thành t
-phép đối xnwgs trục IJ biến DJ thành CJ và Dt thành AC. Khi đó:
$(BA;BJ) \equiv (DJ,t) \equiv (CA,CJ) (mod \pi)$
Từ đây suy ra điều phải chứng minh.
Nhận xét: Qua phép đối xứng trục bài toán trở nên đơn giản hơn rất nhiều.
Blog này tổng hợp các bài toán hay, các bài giảng chọn lọc về nhiều chủ đề: đại số, hình học, giải tích, số học và tổ hợp liên quan đến Toán Olympic và Toán thi ĐH.
Hiển thị các bài đăng có nhãn IMO. Hiển thị tất cả bài đăng
Hiển thị các bài đăng có nhãn IMO. Hiển thị tất cả bài đăng
Thứ Ba, 27 tháng 12, 2016
Chủ Nhật, 23 tháng 10, 2016
IMO shortlist 2002 và những vấn đề liên quan.
Đề bài: Đường tròn nội tiếp (I) của tam giác ABC tiếp xúc với BC tại K. Gọi M là trung điểm đường cao AD. KM cắt (I) tại N. Chứng minh rằng (BCN) tiếp xúc với (I) tại N.
Lời giải:
Đặt $S=KA\cap \Omega$,và đặt $T$ là điểm đối xứng $K$ qua I . Gọi $X,Y$ là các tiếp điểm của (I) trên $CA,AB$.
$AD$ // $KT$ nên $(KT,KN;KS,KD)=-1$. suy ra $KTSN$ điều hòa, vậy tiếp tuyến với $(I)$ tại $K,S$ đồng quy trêb $NT$. Mặt khác, tiếp tuyến tại $X,Y$ đồng quy $KS$, nên $KXSY$ là tứ giác điều hòa, Nghĩa là tiếp tuyến tại $K,S$ đồng quy $XY$.
Từ đây suy ra $BC,XY,TN$ Đồng quy. Gọi $P=XY\cap BC$, suy ra $(B,C;K,P)=-1$,và vì $\angle KNP=\angle KNT=\frac{\pi}2$, nên $NK$ là phân giác của $\angle BNC$.
Gọi $B'=NB\cap \Omega,\ C'=NC\cap \Omega$, Ta có:
$BB^{\prime} \cdot BN = BK^2$
$CC^{\prime} \cdot CN = CK^2$. Suy ra:
$\frac{BB^{\prime} \cdot BN}{CC^{\prime} \cdot CN} = \frac{BK^2}{CK^2}$
Nhưng NK là phân giác của góc BNC nên:
$\frac{BK}{CK} = \frac{BN}{CN}$
Vì vậy:
$\frac{BB^{\prime} \cdot BN}{CC^{\prime} \cdot CN} = \frac{BN^2}{CN^2}$
Suy ra:
$\frac{BB^{\prime}}{CC^{\prime}} = \frac{BN}{CN}$
Nên:
$B'C'\|BC$, nên $N$ là tâm vị tự biến $(I)$ thành $(BNC)$ Hay (I) tiếp xúc (BNC) tại N vậy ta có điều phải chứng minh.
Lời giải:
Đặt $S=KA\cap \Omega$,và đặt $T$ là điểm đối xứng $K$ qua I . Gọi $X,Y$ là các tiếp điểm của (I) trên $CA,AB$.
$AD$ // $KT$ nên $(KT,KN;KS,KD)=-1$. suy ra $KTSN$ điều hòa, vậy tiếp tuyến với $(I)$ tại $K,S$ đồng quy trêb $NT$. Mặt khác, tiếp tuyến tại $X,Y$ đồng quy $KS$, nên $KXSY$ là tứ giác điều hòa, Nghĩa là tiếp tuyến tại $K,S$ đồng quy $XY$.
Từ đây suy ra $BC,XY,TN$ Đồng quy. Gọi $P=XY\cap BC$, suy ra $(B,C;K,P)=-1$,và vì $\angle KNP=\angle KNT=\frac{\pi}2$, nên $NK$ là phân giác của $\angle BNC$.
Gọi $B'=NB\cap \Omega,\ C'=NC\cap \Omega$, Ta có:
$BB^{\prime} \cdot BN = BK^2$
$CC^{\prime} \cdot CN = CK^2$. Suy ra:
$\frac{BB^{\prime} \cdot BN}{CC^{\prime} \cdot CN} = \frac{BK^2}{CK^2}$
Nhưng NK là phân giác của góc BNC nên:
$\frac{BK}{CK} = \frac{BN}{CN}$
Vì vậy:
$\frac{BB^{\prime} \cdot BN}{CC^{\prime} \cdot CN} = \frac{BN^2}{CN^2}$
Suy ra:
$\frac{BB^{\prime}}{CC^{\prime}} = \frac{BN}{CN}$
Nên:
$B'C'\|BC$, nên $N$ là tâm vị tự biến $(I)$ thành $(BNC)$ Hay (I) tiếp xúc (BNC) tại N vậy ta có điều phải chứng minh.
Ta có thêm những kết quả sau:
-NK đi qua tâm bàng tiếp góc A. (Gọi M là trung điểm BC và lưu ý AT đi qua tiếp điểm bàng tiếp trên BC Dùng hàng điểm để chứng minh)
-Nếu gọi $N_B$ tương tự như điểm N ứng với góc B, $N_C$ ứng với góc C thì $NK, N_BX, N_CY$ đồng quy tại S thuộc OI( Dùng cực đối cực)
- Trục đẳng phương của $N_BAC, N_CAB$ và $N_BX, N_CY$ đồng quy tại một điểm thuộc OI. ( ý đầu dùng định lý 4 điểm, ý sau có thể dùng tâm vị tự, cực đối cực) suy ra điểm đồng quy này là S.
Ngoài ra rất nhiều bài toán đẹp có thể được phát triển từ những mô hình này.
Nhãn:
bổ đề hình học,
cực và đối cực,
đồng quy,
đường tròn bàng tiếp,
đường tròn nội tiếp,
hai đường tròn tiếp xúc,
hàng điểm điều hòa,
hình học,
IMO,
IMO 2002
Thứ Ba, 20 tháng 9, 2016
Chứng minh bất đẳng thức trong dãy số
Bài 1: Cho dãy số {$x_n$} xác định bởi $x_n=\frac{1}{n^m(n+a)\sqrt{n+b}} (m>a, m \in Z)$
Khi đó: $x_1 + x_2+..+x_n <\frac{1}{a\sqrt{1+b}}$
Giải:
Ta có:
$x_n=\frac{a}{an^{m-1}\sqrt{n+b}.n(n+a)}=\frac{1}{an^{m-1}\sqrt{n+b}}(\frac{1}{n}-\frac{1}{n+a})$
$x_n=\frac{a}{an^{m-1}\sqrt{n+b}.n(n+a)}=\frac{1}{an^{m-1}\sqrt{n+b}}(\frac{1}{n}-\frac{1}{n+a})=\frac{1}{an^{m}\sqrt{n+b}}-\frac{1}{an^{m-1}(n+a)\sqrt{n+b}}<\frac{1}{an^{m}\sqrt{n+b}}-\frac{1}{a(n+1)^m\sqrt{n+1+b}}$
Đến đây áp dụng công thức tổng sai phân ta có điều phải chứng minh.
Bài 2: Cho dãy số {$u_n$}, { $v_n$} xác định như sau:
$\left\{\begin{matrix}
u_1>0, v_1>0 & & \\
u_{n+1}=u_n+\frac{1}{v_n}, n\ge1 & & \\
v_{n+1}=v_n+\frac{1}{u_n}, n \ge 1& &
\end{matrix}\right.$
Chứng minh rằng: $\sqrt[3]{(u_{2013}+v_{2013})^2} > 2\sqrt[3]{2013}$
Giải:
Đặt $w_n=(u_n+v_n)^2$
$w_{n+1}=(u_{n+1}+v_{n+1})^2=[(u_n+v_n)+(\frac{1}{u_n}+\frac{1}{v_n})]^2>(u_n+v_n)^2+2(u_n+v_n)(\frac{1}{u_n}+\frac{1}{v_n}) \ge w_n+8$
Suy ra: $w_n \ge w_2+(n-2).8$
Chọn n=2013 ta được đpcm
Bài 3: (IMO 1970) Cho dãy số {$U_n$} có tính chất sau:$1=u_o \le u_1 \le ..\le ..$ Xây dựng {$v_n$} như sau:
$v_n=\sum_{k=1}^{n}(1-\frac{u_{k-1}}{u_k})\frac{1}{\sqrt{u_k}} \forall n \ge 1$
a) Chứng minh rằng: $0 \le v_n \le 2$
b) Với mọi số C cho trước$0 \le C <2$ đều tồn tại một dãy số {$u_n$} thỏa mãn điều kiện đã cho và $v_n>C$ với vô số chỉ số n.
Giải:
a)
$v_n$ có dạng dãy tổng nên ta sẽ dùng tổng sai phân:
Theo cách xác định dãy {$v_n$} thì:
$v_n=\sum_{k=1}^{n}(1-\frac{u_{k-1}}{u_k})\frac{1}{\sqrt{u_k}}=2\sum_{k=1}^{n}\frac{u_k-u_{k-1}}{u_k(2\sqrt{u_k})}\\\le2\sum_{k=1}^{n}\frac{u_k-u_{k-1}}{u_k(\sqrt{u_k}+\sqrt{u_{k-1}})}=2\sum_{k=1}^{n}(\frac{1}{\sqrt{u_k}}-\frac{\sqrt{u_{k-1}}}{u_k})\\=2\sum_{k=1}^{n}(\frac{1}{\sqrt{u_k}}+\frac{1}{\sqrt{u_{k-1}}}-(\frac{\sqrt{u_{k-1}}}{u_k}+\frac{1}{\sqrt{u_{k-1}}})) \le 2 \sum_{k=1}^{n} (\frac{1}{\sqrt{u_k}}-\frac{1}{\sqrt{u_{k-1}}})\\=2(1-\frac{1}{\sqrt{u_n}})\le2$
Vậy câu a được chứng minh.
b) Ta sẽ liên tưởng đến lim do điều cần chứng minh:
Bây giờ ta xét dãy số:$u_n=\frac{1}{p^{2n}}$ thỏa mãn điều kiện trên. (p <1)
Đồng thời:$(1-\frac{u_{k-1}}{u_k})\frac{1}{\sqrt{u_k}}=(1-p^2)p^k$ Vì vậy:
$v_n=(1-p^2)\sum_{k=1}^{n}p^k=p(p+1)(1-p^n)=p(p+1)-p^n(p+1)p$
Mà$limp^n=0$ nếu n tới vô cùng. Do vậy ta chỉ cần chứng minh rằng tồn tại q sao cho $C<q<2$ thỏa mãn phương trình x(x+1)=q. Dễ thấy phương trình này có hai nghiệm trai dấu và nghiệm của nó phải thỏa điều kiện $p<1$ nếu không thì$p(p+1) \le 2 >q$.
Mà điều này là hiển nhiên vậy ta có đpcm.
Bài 4 (IMO 1982): Xét dãy số thực dương {$u_n$} thỏa mãn điều kiện:
$1=u_o\ge u_1 \ge u_2.. $
a) Chứng minh rằng với mọi dãy có tính chất như trên đều tồn tại một số $n$ sao cho:
$\frac{u_o^2}{u_1}+\frac{u_1^2}{u_2}+...+\frac{u_{n-1}^2}{u_n} \ge 3,999$
b) Chứng minh tồn tại một dãy như trên mà:
$\frac{u_o^2}{u_1}+\frac{u_1^2}{u_2}+...+\frac{u_{n-1}^2}{u_n} <4$
Giải:
a) Phần chứng minh câu a) làm ta liên tưởng đến cận trên bé nhất của tổng những dãy trên. Vì nếu k là cận trên bé nhất thì $\frac{u_o^2}{u_1}+\frac{u_1^2}{u_2}+...+\frac{u_{n-1}^2}{u_n} >k$
Nhưng $k+ \varepsilon > \frac{u_o^2}{u_1}+\frac{u_1^2}{u_2}+...+\frac{u_{n-1}^2}{u_n} $
Ta sẽ chứng minh $k \ge 4$
Do:
$\frac{u_o^2}{u_1}+\frac{u_1^2}{u_2}+...+\frac{u_{n-1}^2}{u_n}=\frac{u_o^2}{u_1}+u_1(\frac{(\frac{u_1}{u_1})^2}{\frac{u_2}{u_1}}+\frac{(\frac{u_2}{u_1})^2}{\frac{u_3}{u_1}}+..)$
$\Rightarrow k+\varepsilon >\frac{1}{u_1}+u_1k \geq 2\sqrt{k} (\varepsilon >0)$
Đến đây ta cho $ \varepsilon $ càng nhỏ ta có đpcm.
b) Thấy dãy dãy $u_n=2^{-n}$ thỏa mãn bài toán
Bài 5: Cho dãy số {$u_n$} xác định bởi $\left\{\begin{matrix}
u_o=1 & & \\
u_{n+1}=u_n+\frac{1}{u_n} ( \forall n \in N) & &
\end{matrix}\right.$
Chứng minh rằng với mọi số nguyên dương n ta đều có: $\sum_{k=1}^{n}\frac{1}{u_n^4}<\frac{7}{6}$
Nhận xét:
Ta có dãy trên tăng và không bị chặn trên, nên có đánh giá như sau:
$u_{n+1}^2=u_n^2+2+\frac{1}{u_n^2}$ Do $u_n$ tăng và không bị chặn trên nên $\frac{1}{u_n^2}$ sẽ tiến dần tới 0 nên ta sẽ không tính tới.
Mạnh dạn đánh giá: $u_{n+1}^2>u_n^2+2$
Lời giải:
Ta có:
$u_n^2>u_{n-1}^2+2>u_{n-2}^2+4>..u_1^2+2(n-1)=2n+2$
Do đó:
$\frac{1}{u_n^4}<\frac{1}{(2n+2)^2}<\frac{1}{(2n+1)(2n+3)}=\frac{1}{2}(\frac{1}{2n+1}-\frac{1}{2n+3})$
Tới đây cộng các vế ta có đpcm.
Khi đó: $x_1 + x_2+..+x_n <\frac{1}{a\sqrt{1+b}}$
Giải:
Ta có:
$x_n=\frac{a}{an^{m-1}\sqrt{n+b}.n(n+a)}=\frac{1}{an^{m-1}\sqrt{n+b}}(\frac{1}{n}-\frac{1}{n+a})$
$x_n=\frac{a}{an^{m-1}\sqrt{n+b}.n(n+a)}=\frac{1}{an^{m-1}\sqrt{n+b}}(\frac{1}{n}-\frac{1}{n+a})=\frac{1}{an^{m}\sqrt{n+b}}-\frac{1}{an^{m-1}(n+a)\sqrt{n+b}}<\frac{1}{an^{m}\sqrt{n+b}}-\frac{1}{a(n+1)^m\sqrt{n+1+b}}$
Đến đây áp dụng công thức tổng sai phân ta có điều phải chứng minh.
Bài 2: Cho dãy số {$u_n$}, { $v_n$} xác định như sau:
$\left\{\begin{matrix}
u_1>0, v_1>0 & & \\
u_{n+1}=u_n+\frac{1}{v_n}, n\ge1 & & \\
v_{n+1}=v_n+\frac{1}{u_n}, n \ge 1& &
\end{matrix}\right.$
Chứng minh rằng: $\sqrt[3]{(u_{2013}+v_{2013})^2} > 2\sqrt[3]{2013}$
Giải:
Đặt $w_n=(u_n+v_n)^2$
$w_{n+1}=(u_{n+1}+v_{n+1})^2=[(u_n+v_n)+(\frac{1}{u_n}+\frac{1}{v_n})]^2>(u_n+v_n)^2+2(u_n+v_n)(\frac{1}{u_n}+\frac{1}{v_n}) \ge w_n+8$
Suy ra: $w_n \ge w_2+(n-2).8$
Chọn n=2013 ta được đpcm
Bài 3: (IMO 1970) Cho dãy số {$U_n$} có tính chất sau:$1=u_o \le u_1 \le ..\le ..$ Xây dựng {$v_n$} như sau:
$v_n=\sum_{k=1}^{n}(1-\frac{u_{k-1}}{u_k})\frac{1}{\sqrt{u_k}} \forall n \ge 1$
a) Chứng minh rằng: $0 \le v_n \le 2$
b) Với mọi số C cho trước$0 \le C <2$ đều tồn tại một dãy số {$u_n$} thỏa mãn điều kiện đã cho và $v_n>C$ với vô số chỉ số n.
Giải:
a)
$v_n$ có dạng dãy tổng nên ta sẽ dùng tổng sai phân:
Theo cách xác định dãy {$v_n$} thì:
$v_n=\sum_{k=1}^{n}(1-\frac{u_{k-1}}{u_k})\frac{1}{\sqrt{u_k}}=2\sum_{k=1}^{n}\frac{u_k-u_{k-1}}{u_k(2\sqrt{u_k})}\\\le2\sum_{k=1}^{n}\frac{u_k-u_{k-1}}{u_k(\sqrt{u_k}+\sqrt{u_{k-1}})}=2\sum_{k=1}^{n}(\frac{1}{\sqrt{u_k}}-\frac{\sqrt{u_{k-1}}}{u_k})\\=2\sum_{k=1}^{n}(\frac{1}{\sqrt{u_k}}+\frac{1}{\sqrt{u_{k-1}}}-(\frac{\sqrt{u_{k-1}}}{u_k}+\frac{1}{\sqrt{u_{k-1}}})) \le 2 \sum_{k=1}^{n} (\frac{1}{\sqrt{u_k}}-\frac{1}{\sqrt{u_{k-1}}})\\=2(1-\frac{1}{\sqrt{u_n}})\le2$
Vậy câu a được chứng minh.
b) Ta sẽ liên tưởng đến lim do điều cần chứng minh:
Bây giờ ta xét dãy số:$u_n=\frac{1}{p^{2n}}$ thỏa mãn điều kiện trên. (p <1)
Đồng thời:$(1-\frac{u_{k-1}}{u_k})\frac{1}{\sqrt{u_k}}=(1-p^2)p^k$ Vì vậy:
$v_n=(1-p^2)\sum_{k=1}^{n}p^k=p(p+1)(1-p^n)=p(p+1)-p^n(p+1)p$
Mà$limp^n=0$ nếu n tới vô cùng. Do vậy ta chỉ cần chứng minh rằng tồn tại q sao cho $C<q<2$ thỏa mãn phương trình x(x+1)=q. Dễ thấy phương trình này có hai nghiệm trai dấu và nghiệm của nó phải thỏa điều kiện $p<1$ nếu không thì$p(p+1) \le 2 >q$.
Mà điều này là hiển nhiên vậy ta có đpcm.
Bài 4 (IMO 1982): Xét dãy số thực dương {$u_n$} thỏa mãn điều kiện:
$1=u_o\ge u_1 \ge u_2.. $
a) Chứng minh rằng với mọi dãy có tính chất như trên đều tồn tại một số $n$ sao cho:
$\frac{u_o^2}{u_1}+\frac{u_1^2}{u_2}+...+\frac{u_{n-1}^2}{u_n} \ge 3,999$
b) Chứng minh tồn tại một dãy như trên mà:
$\frac{u_o^2}{u_1}+\frac{u_1^2}{u_2}+...+\frac{u_{n-1}^2}{u_n} <4$
Giải:
a) Phần chứng minh câu a) làm ta liên tưởng đến cận trên bé nhất của tổng những dãy trên. Vì nếu k là cận trên bé nhất thì $\frac{u_o^2}{u_1}+\frac{u_1^2}{u_2}+...+\frac{u_{n-1}^2}{u_n} >k$
Nhưng $k+ \varepsilon > \frac{u_o^2}{u_1}+\frac{u_1^2}{u_2}+...+\frac{u_{n-1}^2}{u_n} $
Ta sẽ chứng minh $k \ge 4$
Do:
$\frac{u_o^2}{u_1}+\frac{u_1^2}{u_2}+...+\frac{u_{n-1}^2}{u_n}=\frac{u_o^2}{u_1}+u_1(\frac{(\frac{u_1}{u_1})^2}{\frac{u_2}{u_1}}+\frac{(\frac{u_2}{u_1})^2}{\frac{u_3}{u_1}}+..)$
$\Rightarrow k+\varepsilon >\frac{1}{u_1}+u_1k \geq 2\sqrt{k} (\varepsilon >0)$
Đến đây ta cho $ \varepsilon $ càng nhỏ ta có đpcm.
b) Thấy dãy dãy $u_n=2^{-n}$ thỏa mãn bài toán
Bài 5: Cho dãy số {$u_n$} xác định bởi $\left\{\begin{matrix}
u_o=1 & & \\
u_{n+1}=u_n+\frac{1}{u_n} ( \forall n \in N) & &
\end{matrix}\right.$
Chứng minh rằng với mọi số nguyên dương n ta đều có: $\sum_{k=1}^{n}\frac{1}{u_n^4}<\frac{7}{6}$
Nhận xét:
Ta có dãy trên tăng và không bị chặn trên, nên có đánh giá như sau:
$u_{n+1}^2=u_n^2+2+\frac{1}{u_n^2}$ Do $u_n$ tăng và không bị chặn trên nên $\frac{1}{u_n^2}$ sẽ tiến dần tới 0 nên ta sẽ không tính tới.
Mạnh dạn đánh giá: $u_{n+1}^2>u_n^2+2$
Lời giải:
Ta có:
$u_n^2>u_{n-1}^2+2>u_{n-2}^2+4>..u_1^2+2(n-1)=2n+2$
Do đó:
$\frac{1}{u_n^4}<\frac{1}{(2n+2)^2}<\frac{1}{(2n+1)(2n+3)}=\frac{1}{2}(\frac{1}{2n+1}-\frac{1}{2n+3})$
Tới đây cộng các vế ta có đpcm.
Thứ Ba, 28 tháng 6, 2016
Tiếp tục về tứ giác có hai đường chéo vuông góc
Bài toán: Cho tứ giác ABCD với hai đường chéo AC, BD vuông góc với nhau tại I. Gọi EFGH là tứ giác bàn đạp của I trên AB, BC, CD, DA. Khi đó tâm ngoại tiếp tứ giác EFGH nằm trên đường thẳng nối trung điểm AC và BD.
Lời giải (theo bạn Huỳnh Bách Khoa):
Lời giải (theo bạn Huỳnh Bách Khoa):
Thứ Hai, 27 tháng 6, 2016
Bài toán quốc tế về đường tròn nội tiếp
Đề bài: (IMO 2008) Cho ABCD là tức giác lồi sao cho BA $\ne$ BC. $(I_1 ), (I_2 )$ I
lần lượt là
đường tròn nội tiếp
ABC, ADC .Giả sử tồn tại một đường tròn (I) tiếp xúc với tia
BA,BC và tiếp xúc với AD,CD.Chứng minh rằng :tiếp tuyến chung ngoài của $(I_1 ), (I_2 )$ cắt nhau tại điểm nằm trên (I).
Lời giải:
Gọi giao điểm của $(I_1 ), (I_2 )$ với AC lần lượt là J,L
Ta dễ chứng minh được 2 điều sau :
1) AB+AD=BC+CD. ( Dùng các tính chất của tiếp tuyến cắt nhau cho (I), $(I_1 ), (I_2 )$) )
2)AL=JC. (Suy ra từ 1)
Vẽ các tiếp tuyến của (I), $(I_1 ), (I_2 )$ và song song với AC,các tiếp tuyến này lần lượt tiếp xúc với 3 đường tròn trên tại Z,M,N.
Khi đó, thì ta có:B,M,L,Z (Do JM là đường kính của $(I_1)$, L là tiếp điểm bàng tiếp, và B là tâm của phép vị tự biến I thành $I_1$) và D,N,J,Z thẳng hàng (tương tự) . Lại có:JM và LN song song và là đường kính$(I_1 ), (I_2 )$ ( Chứng minh trùng và lưu ý $I_1, I_2$ và tâm vị tự của chúng thẳng hàng) nên JN và LN cắt nhau tại tâm vị tự ngoài của $(I_1 ), (I_2 )$ . Theo tính chất của tâm vị tự ngoài thì đó là giao điểm hai tiếp tuyến chung ngoài của $(I_1 ), (I_2 )$ nên ta có đpcm.
Lời giải:
1) AB+AD=BC+CD. ( Dùng các tính chất của tiếp tuyến cắt nhau cho (I), $(I_1 ), (I_2 )$) )
2)AL=JC. (Suy ra từ 1)
Vẽ các tiếp tuyến của (I), $(I_1 ), (I_2 )$ và song song với AC,các tiếp tuyến này lần lượt tiếp xúc với 3 đường tròn trên tại Z,M,N.
Khi đó, thì ta có:B,M,L,Z (Do JM là đường kính của $(I_1)$, L là tiếp điểm bàng tiếp, và B là tâm của phép vị tự biến I thành $I_1$) và D,N,J,Z thẳng hàng (tương tự) . Lại có:JM và LN song song và là đường kính$(I_1 ), (I_2 )$ ( Chứng minh trùng và lưu ý $I_1, I_2$ và tâm vị tự của chúng thẳng hàng) nên JN và LN cắt nhau tại tâm vị tự ngoài của $(I_1 ), (I_2 )$ . Theo tính chất của tâm vị tự ngoài thì đó là giao điểm hai tiếp tuyến chung ngoài của $(I_1 ), (I_2 )$ nên ta có đpcm.
Một số tính chất về tam giác ABC có AB+BC=3AC.
Tính chất 1 (IMO shortlist 2005): .Cho tam giác ABC
có AB+BC=3AC.Đường tròn (I) nội tiếp tam giác
tiếp xúc với AB,AC tại D,E.Lấy K,L là điểm đối xứng của D,E qua I.Khi đó ALKC
nội tiếp.
Chứng minh
Gọi H là giao điểm của CK và AB,T là giao điểm của AL và BC.
Ta có các tính chất cơ bản sau:
AH=BD=AC và CT=BE=AC.
$\angle KCA=90^o-\frac{\angle A}{2}=\angle AID$ (Do tam giác AHC cân)
Suy ra tứ giác AKIC nội tiếp
Tương tự tứ giác ALIC nội tiếp.
Hay 5 điểm A, L, K, I, C cùng thuộc đường tròn.
Vậy ta có đpcm.
Tính chất 2 phát biểu: Cho tam giác ABC đường tròn nội tiếp (I), đường tròn bàng tiếp góc A, B, C tương ứng tiếp xúc BC, CA, AB tại M, N, P. Ta có AM, BN, CP đồng quy tại một điểm khi AB+BC=3AC
Ta chứng minh một bổ đề:
Cho tam giác ABC nội tiếp (O), có đường kính AE, BF. Khi đó đường thẳng qua D song song BC và đường thẳng qua E song song AB cắt nhau tại một điểm thuộc (O).
Chứng minh: Gọi hai đường thẳng đó cắt nhau tại F. Do DF song song BC nên DF vuông BD, tương tự EF vuông EB. Vậy tứ giác FDBE nội tiếp hay F thuộc (O).
Trở lại bài toán. ta sẽ lấy hình 1, và dễ thấy M, N, P trong tính chất 2 là E, F, D trong hình trên.
Ta có AL song song EF do có hai tam giác ACT và tam giác CEF cùng cân tại C.
Tương tự CK song song DF. Vậy theo bổ đề trên ta có AL, CK cắt nhau tại 1 điểm thuộc (I).
Ngoài ra ta có bài toán sau: (USA MO 2001)Cho ABC có (I) là đường tròn nội tiếp tiếp xúc với BC,CA lần lượt tại $ D_1, E_1, D_2, E_2,$ là các điểm nằm trên BC,CA sao cho :$C D_2 =B D_1$ và $C E_2 =A E_1$ .Gọi P là giao điểm của $A D_2$ và $B E_2$ .Đường tròn (I) cắt $A D_2$ tại điểm,Q gần A.(Q là giao của $A D_2$ và (I)).Chứng minh :$AQ= D_2 P.$
Lời giải:
Khó có thể chứng minh trực tiếp được nên nếu ta đưa về tỉ số thì sẽ dễ chứng minh hơn. Mà ta lại có:
$\frac{AQ}{AD_2}=\frac{r}{r_a}$ (r là tâm nội tiếp, $r_a $ là tâm bàng tiếp )
Ta sẽ chứng minh : $\frac{D_2P}{AD_2}=\frac{r}{r_a}$
Vì B, P, $E_2$ thẳng hàng.
Theo menelaus cho tam giác $AD_2C$ thì:
$\frac{D_2P}{AP}=\frac{BD_2.CD_1}{BC.E_2A}=\frac{p-a}{a}$
Suy ra:
$\frac{D_2P}{AD_2}=\frac{p-a}{a}=\frac{r}{r_a}$
Vậy ta có đpcm.
Áp dụng bài toán trên thì nếu cho:
P là giao của AT và CH. Thì :HP=CK và AL=PT.
Do đó ta sẽ có: tam giác AKC bằng APH và tam giác CTP bằng tam giác CAL
Từ đây có thể suy ra tính chấtt 1 dễ dàng: $\angle ALC=\angle CPT=\angle APH=\angle AKC$.
Ngoài ra: AK và AT là hai đường đẳng giác trong góc A và CL và CP là hai đường đẳng giác trong góc C. Suy ra L, K là hai điểm đẳng giác của tam giác ABC
Và I là trực tâm APC. Kết hợp với tính chất 2 là P thuộc (I) ta suy ra P đối xứng F qua I.
Chứng minh
Ta có các tính chất cơ bản sau:
AH=BD=AC và CT=BE=AC.
$\angle KCA=90^o-\frac{\angle A}{2}=\angle AID$ (Do tam giác AHC cân)
Suy ra tứ giác AKIC nội tiếp
Tương tự tứ giác ALIC nội tiếp.
Hay 5 điểm A, L, K, I, C cùng thuộc đường tròn.
Vậy ta có đpcm.
Tính chất 2 phát biểu: Cho tam giác ABC đường tròn nội tiếp (I), đường tròn bàng tiếp góc A, B, C tương ứng tiếp xúc BC, CA, AB tại M, N, P. Ta có AM, BN, CP đồng quy tại một điểm khi AB+BC=3AC
Ta chứng minh một bổ đề:
Cho tam giác ABC nội tiếp (O), có đường kính AE, BF. Khi đó đường thẳng qua D song song BC và đường thẳng qua E song song AB cắt nhau tại một điểm thuộc (O).
Chứng minh: Gọi hai đường thẳng đó cắt nhau tại F. Do DF song song BC nên DF vuông BD, tương tự EF vuông EB. Vậy tứ giác FDBE nội tiếp hay F thuộc (O).
Trở lại bài toán. ta sẽ lấy hình 1, và dễ thấy M, N, P trong tính chất 2 là E, F, D trong hình trên.
Ta có AL song song EF do có hai tam giác ACT và tam giác CEF cùng cân tại C.
Tương tự CK song song DF. Vậy theo bổ đề trên ta có AL, CK cắt nhau tại 1 điểm thuộc (I).
Ngoài ra ta có bài toán sau: (USA MO 2001)Cho ABC có (I) là đường tròn nội tiếp tiếp xúc với BC,CA lần lượt tại $ D_1, E_1, D_2, E_2,$ là các điểm nằm trên BC,CA sao cho :$C D_2 =B D_1$ và $C E_2 =A E_1$ .Gọi P là giao điểm của $A D_2$ và $B E_2$ .Đường tròn (I) cắt $A D_2$ tại điểm,Q gần A.(Q là giao của $A D_2$ và (I)).Chứng minh :$AQ= D_2 P.$
Lời giải:
Khó có thể chứng minh trực tiếp được nên nếu ta đưa về tỉ số thì sẽ dễ chứng minh hơn. Mà ta lại có:
$\frac{AQ}{AD_2}=\frac{r}{r_a}$ (r là tâm nội tiếp, $r_a $ là tâm bàng tiếp )
Ta sẽ chứng minh : $\frac{D_2P}{AD_2}=\frac{r}{r_a}$
Vì B, P, $E_2$ thẳng hàng.
Theo menelaus cho tam giác $AD_2C$ thì:
$\frac{D_2P}{AP}=\frac{BD_2.CD_1}{BC.E_2A}=\frac{p-a}{a}$
Suy ra:
$\frac{D_2P}{AD_2}=\frac{p-a}{a}=\frac{r}{r_a}$
Vậy ta có đpcm.
Áp dụng bài toán trên thì nếu cho:
P là giao của AT và CH. Thì :HP=CK và AL=PT.
Do đó ta sẽ có: tam giác AKC bằng APH và tam giác CTP bằng tam giác CAL
Từ đây có thể suy ra tính chấtt 1 dễ dàng: $\angle ALC=\angle CPT=\angle APH=\angle AKC$.
Ngoài ra: AK và AT là hai đường đẳng giác trong góc A và CL và CP là hai đường đẳng giác trong góc C. Suy ra L, K là hai điểm đẳng giác của tam giác ABC
Và I là trực tâm APC. Kết hợp với tính chất 2 là P thuộc (I) ta suy ra P đối xứng F qua I.
Chủ Nhật, 26 tháng 6, 2016
Câu dãy số số học trong IMO và phương pháp suy luận
Đề bài: (IMO 18th) Cho dãy $(u_n)$ xác định như sau: $u_o=2$, $u_1=\frac{5}{2}$ và
$u_{n+1}=u_n(u_{n-1}^2-2)-u_1$
Chứng minh rằng $[u_n]=2^{\frac{2^n-(-1)^n}{3}}$
Lời giải:
Do đề bài yêu cầu chứng minh $[u_n]=2^{\frac{2^n-(-1)^n}{3}}$
Nên ta sẽ cố gắng biểu diễn $u_n$ dưới dang $2^x+a$
Bắt đầu với $u_1$
$u_1=\frac{5}{2}=2+\frac{1}{2}$
$u_2=(2+\frac{1}{2})(2^2-2)-(2+\frac{1}{2})=2+\frac{1}{2}$
$u_3=(2+\frac{1}{2})[(2+\frac{1}{2})^2-2]-(2+\frac{1}{2})$
Ta có thể đoán trước được $u_3=2^3+a (a<1)$ do thay 3 vào điều kiện đề bài
Nên ta tìm cách lấy số $2^3$ ra khỏi $u_3$, ta được:
$u_3=2^3+\frac{1}{2^3}$
Tương tự $u_4=2^5+\frac{1}{2^5}$
Bây giờ ta sẽ chứng minh: $u_n=2^{a_n}+\frac{1}{2^{a_n}}$
Do bài toán cần chứng minh $[u_n]=2^{\frac{2^n-(-1)^n}{3}}$ ta sẽ chứng minh:
Với $a_n=\frac{2^n-(-1)^n}{3}$ đây là công thức tổng quát của $a_n$ tuyến tính bậc 2 nên ta viết lại $a_{n+1}=a_n+2a_{n-1}$
Dễ thấy mệnh đề đúng với n=1,2,3,4,5.
Với n+1 thì:
$u_{n+1}=(2^{a_n}+2^{-a_{n}})(2^{a_{n-1}}+2^{-a_{n-1}-2}-(2+\frac{1}{2})$
Nhân ra ta được:
$u_{n+1}=2^{a_n+2a_{n-1}}+2^{-a_{n}-2a_{n-1}}+2^{2a_{n-1}-a_n}+2^{a_n-2a_{n-1}}-2-2^{-1}$
Mặt khác Do: $a_{n+1}=a_n+2a_{n-1} \Rightarrow 2a_{n-1}-a_n=(-1)^n$
Thay vào ta có đpcm.
$u_{n+1}=u_n(u_{n-1}^2-2)-u_1$
Chứng minh rằng $[u_n]=2^{\frac{2^n-(-1)^n}{3}}$
Lời giải:
Do đề bài yêu cầu chứng minh $[u_n]=2^{\frac{2^n-(-1)^n}{3}}$
Nên ta sẽ cố gắng biểu diễn $u_n$ dưới dang $2^x+a$
Bắt đầu với $u_1$
$u_1=\frac{5}{2}=2+\frac{1}{2}$
$u_2=(2+\frac{1}{2})(2^2-2)-(2+\frac{1}{2})=2+\frac{1}{2}$
$u_3=(2+\frac{1}{2})[(2+\frac{1}{2})^2-2]-(2+\frac{1}{2})$
Ta có thể đoán trước được $u_3=2^3+a (a<1)$ do thay 3 vào điều kiện đề bài
Nên ta tìm cách lấy số $2^3$ ra khỏi $u_3$, ta được:
$u_3=2^3+\frac{1}{2^3}$
Tương tự $u_4=2^5+\frac{1}{2^5}$
Bây giờ ta sẽ chứng minh: $u_n=2^{a_n}+\frac{1}{2^{a_n}}$
Do bài toán cần chứng minh $[u_n]=2^{\frac{2^n-(-1)^n}{3}}$ ta sẽ chứng minh:
Với $a_n=\frac{2^n-(-1)^n}{3}$ đây là công thức tổng quát của $a_n$ tuyến tính bậc 2 nên ta viết lại $a_{n+1}=a_n+2a_{n-1}$
Dễ thấy mệnh đề đúng với n=1,2,3,4,5.
Với n+1 thì:
$u_{n+1}=(2^{a_n}+2^{-a_{n}})(2^{a_{n-1}}+2^{-a_{n-1}-2}-(2+\frac{1}{2})$
Nhân ra ta được:
$u_{n+1}=2^{a_n+2a_{n-1}}+2^{-a_{n}-2a_{n-1}}+2^{2a_{n-1}-a_n}+2^{a_n-2a_{n-1}}-2-2^{-1}$
Mặt khác Do: $a_{n+1}=a_n+2a_{n-1} \Rightarrow 2a_{n-1}-a_n=(-1)^n$
Thay vào ta có đpcm.
Thứ Sáu, 24 tháng 6, 2016
Điều kiện cần và đủ của tứ giác có hai đường chéo vuông góc trong IMO shorlist 2008
Bài: Cho tứ giác lồi ABCD. Chứng minh rằng tồn tại điểm P nằm trong tứ giác thỏa mãn:
$\widehat{PAB}+\widehat{PDC}=\widehat{PBC}+\widehat{PAD}=\widehat{PCD}+\widehat{PBA}=\widehat{PDA}+\widehat{PCB}=90^o$
Khi và chỉ khi hai đường chéo AC và BD vuông góc.
Lời giải:
Giả sử tồn tại P thỏa mãn đề bài. Gọi M là điểm Miquel của tứ giác toàn phần tạo bởi các đường thẳng AB, BC, CD, DA, R là giao BC với DA
Theo điều kiện đề bài:
$180^o=\widehat{PAB}+\widehat{PDC}+\widehat{PCD}+\widehat{PBA}=360-\widehat{BPA}-\widehat{CPD} \Rightarrow \widehat{BPA}=180^o-\widehat{DPC}$
Mặt khác: $\widehat{CRD}=180^o-\widehat{RDC}-\widehat{RCD}=90^o-\widehat{PDC}-\widehat{PCD}=\widehat{CPD}-90^o \Rightarrow \widehat{ARB}=90^o-\widehat{BPA}$
Tương tự ta cũng có: $\widehat{BQC}=90^o-\widehat{BPC}$
Vậy: $\widehat{CPA}=\widehat{BPA}+\widehat{BPC}=180^o-\widehat{BRA}-\widehat{BQC}=180^o-\widehat{BMA}-\widehat{CMB}=180^o-\widehat{CMA}$
Suy ra tứ giác APCM nội tiếp, tương tự DPBM nội tiếp.
$\widehat{BCA}+\widehat{DBC}=\widehat{PCB}-\widehat{PCA}+\widehat{PBC}+\widehat{PBD}=180^o-\widehat{CPB}+\widehat{PMD}-\widehat{DMA}=180^o-\widehat{CPB}-\widehat{AMD}=180^o-\widehat{CPB}-\widehat{DQA}=90^o$
Vậy $AC \perp BD$
Chiều ngược lại chỉ cần chứng minh tồn tại điểm liên hợp đẳng giác với điểm J giao điểm hai đường chéo, trong tứ giác ABCD. Theo lời giải của bạn Huỳnh Bách Khoa
$\widehat{PAB}+\widehat{PDC}=\widehat{PBC}+\widehat{PAD}=\widehat{PCD}+\widehat{PBA}=\widehat{PDA}+\widehat{PCB}=90^o$
Khi và chỉ khi hai đường chéo AC và BD vuông góc.
Lời giải:
Theo điều kiện đề bài:
$180^o=\widehat{PAB}+\widehat{PDC}+\widehat{PCD}+\widehat{PBA}=360-\widehat{BPA}-\widehat{CPD} \Rightarrow \widehat{BPA}=180^o-\widehat{DPC}$
Mặt khác: $\widehat{CRD}=180^o-\widehat{RDC}-\widehat{RCD}=90^o-\widehat{PDC}-\widehat{PCD}=\widehat{CPD}-90^o \Rightarrow \widehat{ARB}=90^o-\widehat{BPA}$
Tương tự ta cũng có: $\widehat{BQC}=90^o-\widehat{BPC}$
Vậy: $\widehat{CPA}=\widehat{BPA}+\widehat{BPC}=180^o-\widehat{BRA}-\widehat{BQC}=180^o-\widehat{BMA}-\widehat{CMB}=180^o-\widehat{CMA}$
Suy ra tứ giác APCM nội tiếp, tương tự DPBM nội tiếp.
$\widehat{BCA}+\widehat{DBC}=\widehat{PCB}-\widehat{PCA}+\widehat{PBC}+\widehat{PBD}=180^o-\widehat{CPB}+\widehat{PMD}-\widehat{DMA}=180^o-\widehat{CPB}-\widehat{AMD}=180^o-\widehat{CPB}-\widehat{DQA}=90^o$
Vậy $AC \perp BD$
Chiều ngược lại chỉ cần chứng minh tồn tại điểm liên hợp đẳng giác với điểm J giao điểm hai đường chéo, trong tứ giác ABCD. Theo lời giải của bạn Huỳnh Bách Khoa
Thứ Năm, 23 tháng 6, 2016
Mở rộng của bài IMO 2005
Đề bài: Cho tứ giác ABCD có DA, BC không song song, P là giao điểm của các đường chéo AC, BD. M, N chạy trên DA, BC sao cho $\dfrac{DM}{DA}=\dfrac{BN}{BC}$. MN theo thứ tự cắt AC, BD tại Q, R. Chứng minh rằng đường tròn (PQR) đi qua một điểm cố định khác P.
Lời giải:
Gọi O là giao điểm của (PAD) và (PBC) thì khi đó:
Xét phép vị tự quay:
$S(O,k, \varphi ):D\rightarrow B\\: A\rightarrow C\\:\Rightarrow DA\rightarrow BC\\:M\rightarrow N(Do:\frac{DM}{AM}=\frac{BN}{NV})$
Suy ra tam giác OAC đồng dạng tam giác OMN, suy ra $\widehat{MNO}=\widehat{QCO}$ Suy ra tứ giác ONQC nội tiếp. Nên O là điểm Miquel của tứ giác toàn phần BPCQNR. Hay O $\in (PQR)$
Lời giải:
Gọi O là giao điểm của (PAD) và (PBC) thì khi đó:
Xét phép vị tự quay:
$S(O,k, \varphi ):D\rightarrow B\\: A\rightarrow C\\:\Rightarrow DA\rightarrow BC\\:M\rightarrow N(Do:\frac{DM}{AM}=\frac{BN}{NV})$
Suy ra tam giác OAC đồng dạng tam giác OMN, suy ra $\widehat{MNO}=\widehat{QCO}$ Suy ra tứ giác ONQC nội tiếp. Nên O là điểm Miquel của tứ giác toàn phần BPCQNR. Hay O $\in (PQR)$
Chủ Nhật, 19 tháng 6, 2016
Vận dụng phương pháp tọa độ kết hợp với phương pháp tổng hợp để giải bài toán hình học
Đôi khi sự kết hợp giữa phương pháp tọa độ và phương phương tổng hợp sẽ giúp cho lời giải ngắn gọn và đẹp hơn. Chúng ta xét ví dụ sau:
Đề bài: (IMO 2000) Cho hai đường tròn $(O_1), (O_2)$ cắt nhau tại M, N. Tiếp tuyến chung gần M của hai đường tròn tiếp xúc $(O_i)$ tại $A_i$, Đường thẳng qua $M$ song song $A_1A_2$ cắt $(O_i) $ở $B_i$, các đường $A_iBi$ cắt nhau tại C, các đường $A_iN$ cắt $B_1B_2$ ở D, E. Chứng minh CD=CE.
Lời giải:
Chọn hệ trục tọa độ $A_1xy$ sao cho $A_1(0;0), A_2(a;0), O_1(0;r_1), O_2(a;r_2)$. Giả sử trong hệ trục $M(s;t)$ Khi đó $B_1(-s;t), B_2(2a-s;t)$ (Tính đối xứng). Từ đó $B_1B_2=2a=2A_1A_2$ Mà $A_1A_2 || B_1B_2$ nên $A_1, A_2$ là trung điểm $B_1C$, $B_2C$, do đó $C(s;-t)$. Vậy $\overrightarrow{CM}=(0;2t),\overrightarrow{B_1B_2}=(2a;0)$, suy ra $CM \perp B_1B_2$, hay $CM \perp DE$
Gọi K là giao MN và $A_1A_2$ ta có:
K là trung điiểm $A_1A_2$ (Dễ cm theo phương tích, trục đẳng phương)
Do $A_1A_2 || B_1B_2$ nên M là trung điểm DE.
Suy ra CM là trung trực của DE. đpcm
Đề bài: (IMO 2000) Cho hai đường tròn $(O_1), (O_2)$ cắt nhau tại M, N. Tiếp tuyến chung gần M của hai đường tròn tiếp xúc $(O_i)$ tại $A_i$, Đường thẳng qua $M$ song song $A_1A_2$ cắt $(O_i) $ở $B_i$, các đường $A_iBi$ cắt nhau tại C, các đường $A_iN$ cắt $B_1B_2$ ở D, E. Chứng minh CD=CE.
Lời giải:
Gọi K là giao MN và $A_1A_2$ ta có:
K là trung điiểm $A_1A_2$ (Dễ cm theo phương tích, trục đẳng phương)
Do $A_1A_2 || B_1B_2$ nên M là trung điểm DE.
Suy ra CM là trung trực của DE. đpcm
Thứ Năm, 28 tháng 4, 2016
Hai bài toán hình học nổi tiếng trong kì thi IMO
Đây là hai bài hình học trong kì thi toán Quốc tế được thầy Trần Quang Hùng đánh giá là đặc biệt và hay trong 7 bài hình học mà thầy đăng
Bài toán 1 (IMO 1985 bài 5 ngày 2). Đường tròn với tâm O đi qua các đỉnh A và C của tam giác ABC cắt các đoạn thẳng BA,BC lần thứ hai tại các điểm K và N. Gọi M là giao điểm của các đường tròn ngoại tiếp tam giác ABC và KBN (khác B). Chứng minh rằng $∠OMB=90^o$.
Bài toán 2 (IMO 1996 bài 2 ngày 1). Cho điểm P nằm trong tam giác ABC sao cho ∠APB−∠ACB=∠APC−∠ABC. Gọi D,E lần lượt là tâm nội tiếp các tam giác APB,APC. Chứng minh rằng AP,BD,CE đồng quy.
Bài toán 1 (IMO 1985 bài 5 ngày 2). Đường tròn với tâm O đi qua các đỉnh A và C của tam giác ABC cắt các đoạn thẳng BA,BC lần thứ hai tại các điểm K và N. Gọi M là giao điểm của các đường tròn ngoại tiếp tam giác ABC và KBN (khác B). Chứng minh rằng $∠OMB=90^o$.
Bài toán 2 (IMO 1996 bài 2 ngày 1). Cho điểm P nằm trong tam giác ABC sao cho ∠APB−∠ACB=∠APC−∠ABC. Gọi D,E lần lượt là tâm nội tiếp các tam giác APB,APC. Chứng minh rằng AP,BD,CE đồng quy.
Tôi xin trình bày tắt lời giải của hai bài toán.
Bài toán 1: Gọi P là giao điểm các đường thẳng AC và KN thì 4 điểm M, P, A, K cùng thuộc một đường tròn (theo định lý Miquel) ngoài ra ta cũng có M nằm trên đoạn BP.
Theo phương tích ta có:
$BO^2-PO^2=BO^2-R^2-(PO^2-R^2)=\overline{BM}.\overline{BP}-\overline{PM}.\overline{PB}=BM^2-PM^2$
Theo định lí 4 điểm ta có điều phải chứng minh.
Bài toán 2: Ta sẽ sử dụng bổ đề: Cho P là một điểm năm trong tam giác ABC. X,Y,Z lần lượt là chân đường vuông góc hạ từ P xuống BC, CA, AB.
Khi đó:$ PA=\dfrac{YZ}{sin A}$ và $\angle APB-\angle ACB=\angle XZY$
Bổ đề này xin dành cho bạn đọc chứng minh.
Trở lại bài toán. Ta có: $\angle APB-\angle ACB=\angle XZY$ và $\angle APC-\angle ABC=\angle XYZ$
Suy ra tam giác XYZ cân, với XY=XZ. Từ đó:
$PCsinACB=PB.sin ABC$ dùng thêm định lí hàm sin ta được
$\dfrac{AB}{PB}=\dfrac{AC}{PC}$
Nếu BD cắt AP tại W thì theo định lí Stewwart ta có $\dfrac{AB}{PB}=\dfrac{AC}{PC}=\dfrac{AW}{PW}$ và CW cũng là phân giác, ta có điều phải chứng minh.
Đăng ký:
Bài đăng (Atom)
Bất đẳng thức tuyển sinh lớp 10 chọn lọc
Trong bài viết này, tác giả giới thiệu một số bài BĐT nhẹ nhàng nhưng ý tưởng tương đối mới, mức độ phù hợp với đề thi tuyển sinh vào lớp...
-
I) Hàm phần nguyên: 1) Định nghĩa Phần nguyên của một số thực x là số nguyên lớn nhất không vượt quá x. Kí hiệu là [x]. 2) Tính chất...
-
Định nghĩa 1: Một số nguyên a được gọi là thặng dư bình phương mod n nếu tồn tại số nguyên x sao cho $x^2 \equiv a (mod n)$ Ta cũng có th...
-
Trong thế giới bất đẳng thức , ngoài những bất đẳng thức kinh điển và được áp dụng rất nhiều như bất đẳng thức AM – GM, bất đẳng thức Cauc...